Skip to main content
Log in

The second weight of generalized Reed-Muller codes in most cases

  • Published:
Cryptography and Communications Aims and scope Submit manuscript

Abstract

The second weight of the Generalized Reed-Muller code of length q n and order d over the finite field with q elements is now known for d < q and d > (n − 1)(q − 1). In this paper, we determine the second weight for the other values of d which are not multiples of q − 1 plus 1. For the special case d = a(q − 1) + 1 we give an estimate.

This is a preview of subscription content, log in via an institution to check access.

Access this article

Price excludes VAT (USA)
Tax calculation will be finalised during checkout.

Instant access to the full article PDF.

Institutional subscriptions

Similar content being viewed by others

References

  1. Cherdieu, J.-P., Rolland, R.: On the number of points of some hypersurfaces in \({{\mathbb{F}}}_q^n\). Finite Fields Their Appl. 2, 214–224 (1996)

    Article  MATH  MathSciNet  Google Scholar 

  2. Cox, D., Little, J., O’Shea, D.: Ideals, Varieties, and Algorithms, 2nd edn. Springer, New York (1997)

    Google Scholar 

  3. Delsarte, P., Goethals, J., MacWilliams, F.: On generalized Reed-Muller codes and their relatives. Inf. Control 16, 403–442 (1970)

    Article  MATH  MathSciNet  Google Scholar 

  4. Dickson, L.: Linear Groups. Dover, New York (1958) (Reprint of L. Dickson, Linear Groups, B.G. Teubner, Leipzig (1901))

    MATH  Google Scholar 

  5. Geil, O.: On the second weight of generalized Reed-Muller codes. Designs Codes Cryptogr. 48(3), 323–330 (2008)

    Article  MATH  MathSciNet  Google Scholar 

  6. Geil, O.: On codes from norm-trace curves. Finite Fields Their Appl. 9, 351–371 (2003)

    Article  MATH  MathSciNet  Google Scholar 

  7. Kasami, T., Lin, S., Peterson, W.: New generalizations of the Reed-Muller codes part I: primitive codes. IEEE Trans. Inf. Theory IT-14(2), 189–199 (1968)

    Article  MathSciNet  Google Scholar 

  8. Kasami, T., Tokura, N., Azumi, S.: On the Weight Enumeration of Weights Less than 2.5d of Reed-Muller Codes. Faculty of Engineering Science, Osaka University, Osaka Japan (1974)

    Google Scholar 

  9. McEliece, R.: Quadratic Forms over Finite Fields and Second-order Reed-Muller Codes. JPL Space Programs Summary, vol. III, pp. 37–58 (1969)

  10. Rolland, R.: Number of points of non-absolutely irreducible hypersurfaces. In: Algebraic Geometry and its Applications, Proceedings of the First SAGA Conference, 7–11 May 2007. Papeete, Number Theory and Its Applications, vol. 5, pp. 481–487. World Scientific, Singapore (2008)

    Google Scholar 

  11. Sboui, A.: Second highest number of points of hypersurfaces in \({{\mathbb{F}}}_q^n\). Finite Fields Their Appl. 13(3), 444–449 (2007)

    Article  MATH  MathSciNet  Google Scholar 

Download references

Author information

Authors and Affiliations

Authors

Corresponding author

Correspondence to Robert Rolland.

Appendix A: Proof of lemma 3.10

Appendix A: Proof of lemma 3.10

1.1 Preliminary remarks

Let us set

$$ P_1=\prod\limits_{i=1}^n (q-\alpha_i), $$
$$ P_2=(q-\gamma)\prod\limits_{i=a+2}^n (q-\alpha_i). $$

Hence we have to study the minimum value of P 1 − P 2. Note that in the particular case d = (n − 1)(q − 1) the value of a is n − 1 and P 2 = (q − γ).

Lemma A.1

If we permute the first a elements α i we don’t change the value of P1 − P2. When αa + 1 < b, if we permute the last n − a − 1 elements we don’t change the value of P1 − P2. When αa + 1 ≥ b, namely when γ = αa + 1, if we permute αa + 1 with one of the last n − a − 1 elements α i such that α i  ≥ b we don’t change the value of P1 − P2.

Proof

This can be seen directly on the formulas giving P 1 and P 2.□

Then, from now on, we will suppose that the sequences α are such that

$$ \alpha_1 \geq \alpha_2 \geq \cdots \geq \alpha_{a-1} \geq \alpha_a, $$
$$ \alpha_{a+2} \geq \alpha_{a+3} \geq \cdots \geq \alpha_{n} $$

if α a + 1 < b, and that

$$ \alpha_1 \geq \alpha_2 \geq \cdots \geq \alpha_{a-1} \geq \alpha_a, $$
$$ \alpha_{a+1} \geq \alpha_{a+2} \geq \cdots \geq \alpha_{n} $$

if α a + 1 ≥ b. In particular, when we transform a sequence, we always reorder the new obtained sequence in this way.

Lemma A.2

If we replace α i by α i  + 1 and if the new sequence is in V, then the new P1 − P2 is lower than the old one.

Proof

When i ≤ a the value of P 1 decreases, the value of P 2 is not modified. When i = a + 1 and α a + 1 < b, P 1 decreases, the value of P 2 is not modified. When i ≥ a + 1 and α a + 1 ≥ b, P 2 and P 1 decreases, then we must examine more precisely the behaviour of P 1 − P 2. The difference between the old value of P 1 − P 2 and the new one is

$$\begin{array}{lll} &&\prod\limits_{\begin{array}{c} j=1\\j\neq i \end{array}}^n \left ( q-\alpha_j\right) - \prod\limits_{\begin{array}{c} j=a+1\\j\neq i \end{array}}^n \left ( q-\alpha_j\right) \\ && \;\; = \left ( \prod\limits_{j=1}^a \left ( q-\alpha_j\right)-1 \right ) \prod\limits_{\begin{array}{c} j=a+1\\j\neq i \end{array}}^n \left ( q-\alpha_j\right). \end{array}$$

But, as α a + 1 ≥ b, α a  ≤ q − 2 and then

$$ \prod\limits_{j=1}^a \left ( q-\alpha_j\right) \geq 2. $$

We conclude that the new value is lower than the old one. It remains to study the case where α a + 1 < b and i > a + 1. The difference between the old value of P 1 − P 2 and the new one is now

$$ \left ( \prod\limits_{j=1}^{a+1} \left ( q-\alpha_j\right)-(q-b) \right ) \prod\limits_{\begin{array}{c} j=a+2\\j\neq i \end{array}}^n \left ( q-\alpha_j\right). $$

But as (q − α a + 1) > (q − b) we conclude that the new value of P 1 − P 2 is lower than the old one. □

Lemma A.3

The minimum in the equation (1) is reached for sequences α such that \(\sum_{i=1}^n \alpha_i=K\).

Proof

It is sufficient to prove that if \(\sum_{i=1}^n \alpha_i<K\) it is possible to add 1 to a well chosen α i (and then increase the sum), and obtain a new sequence in V for which the new P 1 − P 2 is lower than the old one. Suppose that \(\sum_{i=1}^n \alpha_i<K\).

  1. 1)

    Suppose that α a + 1 ≥ b.

    1. a)

      If (α 1,...,α a ) ≠ (q − 1,...,q − 1,q − 2) then there exists an index i such that 1 ≤ i ≤ a and such that α i does not reach its maximal value. Then we can replace α i by α i  + 1. By Lemma A.2 we conclude that the new P 1 − P 2 is lower than the old one.

    2. b)

      Now suppose that (α 1,...,α a ) = (q − 1,...,q − 1,q − 2).

      1. α)

        If b = 0 then K = d + 1 = a(q − 1) + 1. But the sum of the first a elements is a(q − 1) − 1. So that α a + 1 is at most 1 (this term exists because a ≤ n − 1). Then we can replace α a + 1 by α a + 1 + 1 because q ≥ 3. By Lemma A.2 we conclude that the new P 1 − P 2 is lower than the old one.

      2. β)

        If b ≥ 1 then K = d + q − b = a(q − 1) + q. In this case we know that a ≤ n − 2. We have α a + 1 + α a + 2 + ...α n  ≤ q then if α a + 1 < q − 1 we can add 1 to this term, if α a + 1 = q − 1 then α a + 2 ≤ 1 and because q ≥ 3 it is possible to add 1 to to this term. By Lemma A.2 we conclude that the new P 1 − P 2 is lower than the old one.

  2. 2)

    Suppose that α a + 1 < b. Then by Lemma A.2 if we replace α a + 1 by α a + 1 + 1, we obtain a new P 1 − P 2 lower than the old one.

From now on we will suppose that α is such that

$$ \sum\limits_{i=1}^n \alpha_i=K $$

Lemma A.4

Let 1 ≤ i ≤ a and a + 1 ≤ j ≤ n and suppose that α j  > α i . If we permute these two elements, and if we obtain a sequence which is in V, then for the new sequence the value of P1 − P2 is lower or equal to the old one.

Proof

Indeed P 1 does not change, and P 2 increases (if j > a + 1 or if j = a + 1 and α j  > b) or does not change (if j = a + 1 and α j  ≤ b).□

Lemma A.5

Suppose that 1 ≤ α i  ≤ α j  ≤ q − 2 and that we are in one of the following cases:

  1. (1)

    1 ≤ j < i ≤ a;

  2. (2)

    a + 2 ≤ j < i ≤ n;

  3. (3)

    αa + 1 ≥ b and a + 1 ≤ j < i ≤ n;

  4. (4)

    1 ≤ j ≤ a and a + 2 ≤ i ≤ n.

Let us replace α i by α i  − 1 and α j by α j  + 1. If the new sequence is in V, the new value of P1 − P2 is lower than the old one.

Proof

  1. 1)

    Case 1 ≤ j < i ≤ a. The difference between the old value of P 1 − P 2 and the new value is

    $$ (\alpha_j-\alpha_i+1) \prod\limits_{k \neq i,j}(q-k) > 0. $$
  2. 2)

    Case a + 2 ≤ j < i ≤ n. The difference between the the old value of P 1 − P 2 and the new value is

    $$ (\alpha_j-\alpha_i+1) \left ( \prod\limits_{k=1}^{a+1} \left ( q-\alpha_k\right)-(q-\gamma) \right ) \prod\limits_{\begin{array}{c} k=a+2\\k\neq i,j \end{array}}^n \left ( q-\alpha_k\right) >0 . $$

    To verify that the previous expression is > 0 note that if α a + 1 < b then γ = b and

    $$ \prod\limits_{k=1}^{a+1} \left ( q-\alpha_k\right) \geq (q-\alpha_{a+1}). $$

    Hence

    $$ \prod\limits_{k=1}^{a+1} \left ( q-\alpha_k\right)-(q-b)\geq (b-\alpha_{a+1})>0. $$

    If α a + 1 ≥ b then γ = α a + 1 and α a  ≤ q − 2. Then

    $$ \prod\limits_{k=1}^{a+1} \left ( q-\alpha_k\right)-(q-\gamma) \geq 2(q-\alpha_{a+1}) - (q-\alpha_{a+1})=(q-\alpha_{a+1}) >0. $$
  3. 3)

    Case α a + 1 ≥ b and a + 1 ≤ j < i ≤ n. The formula of the difference between the the old value of P 1 − P 2 and the new value is similar

    $$ (\alpha_j-\alpha_i+1) \left ( \prod\limits_{k=1}^{a} \left ( q-\alpha_k\right)-1 \right ) \prod\limits_{\begin{array}{c} k=a+1\\k\neq i,j \end{array}}^n \left ( q-\alpha_k\right) > 0. $$

    To verify that the previous expression is > 0 we have just to remark that α a  ≤ q − 2, then \(\prod_{k=1}^{a}\left ( q-\alpha_k\right)\geq 2\).

  4. 4)

    Case 1 ≤ j ≤ a and a + 2 ≤ i ≤ n. A simple computation shows that the difference between the old value of P 1 − P2 and the new value is

    $$ (\alpha_j-\alpha_i+1)\prod\limits_{\begin{array}{c} k=1\\k\neq i,j \end{array}}^n \left ( q-\alpha_k\right)\quad +\quad (q-\gamma) \prod\limits_{\begin{array}{c} k=a+2\\\ k\neq i \end{array}}^n \left ( q-\alpha_k\right) >0 $$

1.2 A.2 The head of a best sequence

We give here the form of the first a terms of a sequence α for which P 1 − P 2 is minimum. We prove that α can be chosen such that one of the two following conditions holds:

  1. (1)

    (α 1,...α a − 1, α a ) = (q − 1,...,q − 1,q − 2) and α a + 1 ≥ b;

  2. (2)

    (α 1,..., α a ) = (q − 1,...,q − 1) and α a + 1 < b;

  1. 1)

    Let us suppose first that there exists a sequence α such that α a + 1 < b and for which P 1 − P 2 is minimum. We will prove that for such a sequences the first a terms can be set to q − 1. Suppose that there exists a j ≤ a such that α j  < q − 1. We have

    $$ \sum\limits_{i=1}^{a+1} \alpha_i <a(q-1)+b=d, $$

    then α a + 2 > 0. If α a + 2 > α j by Lemma A.4 we can permute the two terms to obtain a sequence which have a lower or equal P 1 − P 2. If α a + 2 ≤ α j , by Lemma A.5 the sequence obtained by replacing α a + 2 by α a + 2 − 1 and α j by α j  + 1 has a lower P 1 − P 2. So we have proved that we can increase the value of α j and obtain a lower or equal P 1 − P 2.

  2. 2)

    Let us suppose now that there exists a sequence α, such that α a + 1 ≥ b, for which P 1 − P 2 is minimum. We will prove that for such a sequence the first a − 1 terms can be set to q − 1 and α a can be set to q − 2. Suppose that there exists a j ≤ a such that α j  < q − 1 if j < a or or that α j  < q − 2 if j = a.

    1. a)

      If b = 0 then K = d + 1 = a(q − 1) + 1. But \(\sum_{i=1}^{a} \alpha_i < a(q-1)-1\). Then α a + 1 > 0. If α a + 1 > α j by Lemma A.4 we can permute the two terms to obtain a sequence which have a lower or equal P 1 − P 2. If α a + 1 ≤ α j , by Lemma A.5 the sequence obtained by replacing α a + 1 by α a + 1 − 1 and α j by α j  + 1 has a lower P 1 − P 2. So we have proved that we can increase the value of α j and obtain a lower or equal P 1 − P 2.

    2. b)

      If b > 0 then K = d + q − b = a(q − 1) + q. But \(\sum_{i=1}^{a} \alpha_i < a(q-1)-1+\) and then \(\sum_{i=1}^{a+1} \alpha_i < a(q-1)+q-2+\). Hence α a + 2 > 0. With the same method than in the previous part 1) we prove that we can increase the value of α j and obtain a lower or equal P 1 − P 2.

1.3 A.3 The tail of a best sequence

We give here the form of the terms α i for i ≥ a + 1 of a sequence α for which P 1 − P 2 is minimum, assuming that the head is as in the previous subsection.

  1. 1)

    Let us suppose first that there exists a sequence α such that α a + 1 < b and for which P 1 − P 2 is minimum. We have seen in the previous subsection that we can suppose that the first a terms are q − 1. We know that K = a(q − 1) + q. Then \(\sum_{i=a+2}^{n} \alpha_i=q-\alpha_{a+1}\) using Lemma A.5 we can pack the terms α i for i ≥ a + 2 in such a way that

    1. a)

      if α a + 1 = 0 then α a + 2 = q − 1, α a + 3 = 1 and α i  = 0 for i > a + 3;

    2. b)

      if b > α a + 1 ≥ 1 then α a + 2 = q − α a + 1 and and α i  = 0 for i > a + 2.

  2. 2)

    Let us suppose now that α a + 1 ≥ b. We We have seen in the previous subsection that we can suppose that the a − 1 first a − 1 terms are q − 1 and α a  = q − 2.

    1. a)

      If b = 0 then K = a(q − 1) + 1. Then by Lemma A.5 we can pack the terms α i for i ≥ a + 1 in such a way that α a + 1 = 2 and α i  = 0 for i > a + 1.

    2. b)

      If b > 0 then K = a(q − 1) + q. Then by Lemma A.5 we can pack the terms α i for i ≥ a + 1 in such a way that α a + 1 = q − 1, α a  + 2 = 2 and α i  = 0 for i > a + 2.

1.4 A.4 The minimum value of P 1 − P 2

  1. 1)

    Case b=0. Then K = a(q − 1) + 1. The previous results give directly a sequence for which P 1 − P 2 is minimum:

    $$ \alpha_1=\cdots =\alpha_{a-1}= q-1, $$
    $$ \alpha_a=q-2, \quad \alpha_{a+1}=2, \quad \alpha_{a+2}=\cdots\alpha_n=0. $$

    For this sequence we have

    $$ P_1=2(q-2)q^{n-a-1} \quad P_2= (q-2)q^{n-a-1}, $$

    then the minimum value of P 1 − P 2 is

    $$ \mu = (q-2)q^{n-a-1}. $$
  2. 2)

    Case b=1. Then K = a(q − 1) + q and a ≤ n − 2.

    1. a)

      Let us test first the assumption α a + 1 = 0. The previous results give directly a sequence reaching the minimum of P 1 − P 2 under this assumption:

      $$ \alpha_1=\cdots =\alpha_{a}= q-1, $$
      $$ \alpha_{a+1}=0,\quad \alpha_{a+2}=q-1, \quad \alpha_{a+3}=1, \quad \alpha_{a+4}=\cdots\alpha_n=0. $$

      We remark that if a = n − 2 this case cannot occur because there is not enough room to contain all the α i . For this sequence we have

      $$ P_1=q(q-1)q^{n-a-3},\quad P_2=(q-1)(q-1)q^{n-a-3}, $$

      so that the minimum of P 1 − P 2 under this assumption is

      $$ \mu_1=(q-1)q^{n-a-3}. $$
    2. b)

      Now let us test the assumption α a + 1 ≥ b = 1. The previous results give directly a sequence reaching the minimum of P 1 − P 2 under this assumption:

      $$ \alpha_1=\cdots =\alpha_{a-1}= q-1, $$
      $$ \alpha_{a}=q-2, \quad \alpha_{a+1}=q-1,\quad \alpha_{a+2}=2, \quad \alpha_{a+3}=\cdots\alpha_n=0. $$

      For this sequence we have

      $$ P_1=2(q-2)q^{n-a-2},\quad P_2=(q-2)q^{n-a-2}, $$

      so that the minimum of P 1 − P 2 under this assumption is

      $$ \mu_2=(q-2)q^{n-a-2}. $$
    3. c)

      Conclusion on the case b = 1. Let us compare μ 1 and μ 2 (when a < n − 2)):

      $$ \mu_2-\mu_1=q^{n-a-1}-2q^{n-a-2}-q^{n-a-2}+q^{n-a-3}, $$
      $$ \mu_2-\mu_1=q^{n-a-2}(q-3)+q^{n-a-3}. $$

      But q ≥ 3, then μ 2 > μ 1. Hence the minimum value is μ 1.

      Let us summarize the obtained result in the case b = 1:

      • if a < n − 2 then \(\mu=\mu_1=(q-1)q^{n-a-3}\);

      • if a = n − 2 then \(\mu=\mu_2=(q-2)q^{n-a-2}=q-2\).

  3. 3)

    Case 2 ≤ b < q − 1. Then K = a(q − 1) + q and a ≤ n − 2.

    1. a)

      Test of the assumption α a + 1 < b.

      1. α)

        Test of the joint assumption α a + 1 = 0. The previous results give directly a sequence reaching the minimum of P 1 − P 2 under this assumption:

        $$ \alpha_1=\cdots =\alpha_{a}= q-1, $$
        $$ \alpha_{a+1}=0,\quad \alpha_{a+2}=q-1, \quad \alpha_{a+3}=1, \quad \alpha_{a+4}=\cdots\alpha_n=0. $$

        This case cannot occur if a = n − 2. For this sequence we have

        $$ P_1=q(q-1)q^{n-a-3},\quad P_2=(q-b)(q-1)q^{n-a-3}. $$

        Then the minimum reached by P 1 − P 2 under this assumption is

        $$ \mu_1=b(q-1)q^{n-a-3}. $$
      2. β)

        Test of the joint assumption α a + 1 ≠ 0. The previous results shows that a sequence reaching the minimum of P 1 − P 2 under these assumptions is of the form

        $$ \alpha_1=\cdots =\alpha_{a}= q-1, $$
        $$ \alpha_{a+1}>0,\quad \alpha_{a+2}=q-\alpha_{a+1}, \quad \alpha_{a+3}=\cdots\alpha_n=0. $$

        For this sequence we have

        $$ P_1=(q-\alpha_{a+1})\alpha_{a+1}q^{n-a-2}, \quad P_2=(q-b)\alpha_{a+1}q^{n-a-2}, $$

        then

        $$ P_1-P_2=(b-\alpha_{a+1})\alpha_{a+1}q^{n-a-2}. $$

        The minimum of the quadratic polynomial (b − α a + 1)α a + 1 (with 1 ≤ α a + 1 < b ≤ q − 2) is reached for α a + 1 = 1 which gives for minimum of P 1 − P 2

        $$ \mu_2=(b-1)q^{n-a-2}. $$
    2. b)

      Test of the assumption α a + 1 ≥ b. The previous results shows that a sequence reaching the minimum of P 1 − P 2 under this assumption is

      $$ \alpha_1=\cdots =\alpha_{a-1}= q-1, $$
      $$ \alpha_{a}=q-2, \quad \alpha_{a+1}=q-1,\quad \alpha_{a+2}=2, \quad \alpha_{a+3}=\cdots\alpha_n=0. $$

      For this sequence we have

      $$ P_1=2(q-2)q^{n-a-2},\quad P_2=(q-2)q^{n-a-2}, $$

      so that the minimum of P 1 − P 2 under this assumption is

      $$ \mu_3=(q-2)q^{n-a-2}. $$
    3. c)

      Conclusion of the case 2 ≤ b < q − 1. The minimum of P 1 − P 2 is

      $$ \mu =\min(\mu_1,\mu_2,\mu_3)=\mu_2=(b-1)q^{n-a-2}. $$

      Indeed, as q − 1 > b ≥ 2, we have q − 2 > b − 1 > 0, which prove that μ 3 > μ 2. To prove that μ 1 > μ 2 let us compute

      $$ \mu_1-\mu_2=b(q-1)q^{n-a-3}-(b-1)q^{n-a-2} = q^{n-a-2}-b\!q^{n-a-3}. $$

      But b < q, then μ 1 − μ 2 > 0.

Rights and permissions

Reprints and permissions

About this article

Cite this article

Rolland, R. The second weight of generalized Reed-Muller codes in most cases. Cryptogr. Commun. 2, 19–40 (2010). https://doi.org/10.1007/s12095-009-0014-2

Download citation

  • Received:

  • Accepted:

  • Published:

  • Issue Date:

  • DOI: https://doi.org/10.1007/s12095-009-0014-2

Keywords

Mathematics Subject Classifications (2000)

Navigation